0



Hat hier jemand eventuell eine Lösung / einen Ansatz dazu?
Diese Frage melden
gefragt

Punkte: 10

 
Kommentar schreiben
1 Antwort
0

Ich würde folgenden Ansatz wählen:
Kostenfunktion: 10000*K +5000*L soll minimal werden.
Nebenbedingung: Produktionsfunktion \(q=4K^{0.5} +L^{0,5}\).
Lagrange: Weil das L schon durch die Variable Lohn belegt ist , nehme ich\(\Lambda\)  als Bezeichnung für die Lagrangefunktion
\(\Lambda =10000*K +5000*L +\lambda(q-4K^{0,5}-L^{0,5})\)
\({\partial \Lambda \over \partial K}=10.000 -\lambda * {4  \over 2 K^{0,5}}=0\)
\({\partial \Lambda \over \partial L}=5000 -\lambda *{1 \over 2L^{0,5}}=0\)
\({\partial \Lambda \over \partial \lambda} =q-4K^{0,5}-L^{0,5}=0\)
Aus den beiden oberen Gleichungen folgt : \(10000 -\lambda{4 \over 2K^{0,5}}= 5000 -\lambda*{1 \over 2L^{0,5}} \Rightarrow 2={{2*\lambda \over K^{0,5}} \over {\lambda \over 2L^{0,5}} } = {4L^{0,5} \over K^{0,5}}\Rightarrow {1 \over 2} =({L \over K})^{0,5} \Rightarrow {L \over K}={1 \over4} =0,25 \)
(ohne Garantie für richtiges Rechnen)

Diese Antwort melden
geantwortet

Sonstiger Berufsstatus, Punkte: 12.68K

 

Kommentar schreiben